LSAT and Law School Admissions Forum

Get expert LSAT preparation and law school admissions advice from PowerScore Test Preparation.

 Administrator
PowerScore Staff
  • PowerScore Staff
  • Posts: 8917
  • Joined: Feb 02, 2011
|
#40055
Complete Question Explanation
(The complete setup for this game can be found here: lsat/viewtopic.php?t=6800)

The correct answer choice is (C)

If S is the fourth piece, then V must be the fifth, because S must be earlier than V (from the first rule). We can use a linear setup to represent the fixed placement of these two variables:
PT73_Game_#1_#2_diagram 1.png
To establish the relationship between the remaining three variables, recall that T can never be between R and S (second rule). Since S is fourth, we must ensure that the following sequence does not occur:
  • ..... ..... ..... ..... ..... R :longline: T :longline: S
Accordingly, T must be earlier than R:
PT73_Game_#1_#2_diagram 2.png
At this point, it would be wise to take a look at the answer choices and check if any of them can be validated by the inferences made so far. And indeed, answer choice (C) must be true (T R) and is therefore correct. Granted, the setup above does not take into account the third rule of the game, which restricts the placement of W relative to T and R. Nevertheless, observant test takers would not even bother with that rule, because its application cannot alter the fundamental deduction that T is earlier than R. Thus, the following two local diagrams, which take into account the restrictions placed by the third rule, are presented for pedagogical purposes only:
PT73_Game_#1_#2_diagram 3.png
Of course, if you had the four sequencing templates discussed earlier, you would not even need to construct either of these local diagrams. This is because the local condition (S is fourth) confines any possible solutions to Templates 1A or 1B. In either template, T must be earlier than R, which immediately proves answer choice (C) correct.
Answer choice (A) is not necessarily true, because W could be earlier than R. See Template 1A, or the first local diagram above.

Answer choice (B) cannot be true, because T must be earlier than S.

Answer choice (C): This is the correct answer choice, as discussed above.

Answer choice (D) cannot be true, because V must be the last piece on the CD.

Answer choice (E) is attractive, but incorrect. While W could be earlier than both T and R, it is also possible that W is later than both T and R, in accordance with the third rule:
PT73_Game_#1_#2_diagram 4.png
This answer choice can also be eliminated by reference to Template 1B.
You do not have the required permissions to view the files attached to this post.

Get the most out of your LSAT Prep Plus subscription.

Analyze and track your performance with our Testing and Analytics Package.